Find a linear fractional transformation that carries circle to a line.

Click For Summary
The discussion focuses on finding a linear fractional transformation that maps the unit circle to the line defined by Re((1 + w)) = 0, which represents the vertical line y = x. The transformation is defined using three unique points, with the user selecting points 1, i, and -1 to map to infinity, 0, and 1+i, respectively. The user attempts to derive the transformation L but encounters difficulties simplifying their calculations to match the book's answer. Clarification is provided regarding the mapping points and the correct interpretation of the line equation. The conversation emphasizes the need to cancel terms involving infinity and correctly identify the transformation's structure.
StumpedPupil
Messages
11
Reaction score
0

Homework Statement



Define L: |z| = 1 -----> Re( (1 + w)) = 0. Find L.

Homework Equations



A transformation is defined by three unique points by T(z) = (z-z1)(z2-z3) / (z-z3)(z2-z1). If we have two transformations T and S, and we want T = S for three distinct points, then we have the transformation L by the transformation S^(-1)[T(z)].

The Attempt at a Solution



I chose the points on the circle 1, i, and -1 to go to the points infinity, 0 and 1+i respectively. My calculations gave me L(z) = (1+i)((z-i) - (z+1))(infinity) / ((infinity)(z-1)i - (z+1)(1+i)). The book gives me u(1-i)(z+1)/(z-1) where u is any real number.

What should I do to get this simple form (aka the right answer). Thank you.

 
Physics news on Phys.org
Why are you mapping points to 0 and 1 + i? Doesn't Re(1 + w) = 0 represent a vertical line?

Cancel out the terms containing infinity.
 
I'm sorry, that is a typo. I meant to write Re((1 + i)w) = 0. This is the line y = x. I choose two points on this line, and a point at infinity.
 
Question: A clock's minute hand has length 4 and its hour hand has length 3. What is the distance between the tips at the moment when it is increasing most rapidly?(Putnam Exam Question) Answer: Making assumption that both the hands moves at constant angular velocities, the answer is ## \sqrt{7} .## But don't you think this assumption is somewhat doubtful and wrong?

Similar threads

  • · Replies 3 ·
Replies
3
Views
2K
  • · Replies 3 ·
Replies
3
Views
2K
  • · Replies 2 ·
Replies
2
Views
2K
  • · Replies 2 ·
Replies
2
Views
1K
Replies
1
Views
1K
  • · Replies 8 ·
Replies
8
Views
2K
  • · Replies 3 ·
Replies
3
Views
2K
  • · Replies 2 ·
Replies
2
Views
3K
Replies
9
Views
2K
  • · Replies 1 ·
Replies
1
Views
1K